8 svar
2097 visningar
Nide är nöjd med hjälpen
Nide 114
Postad: 1 okt 2018 16:29

Avgöra om generaliserad integral konvergerar eller divergerar

Ska avgöra om integralen 0x3+1x5+x3+1dx divergerar eller konvergerar.

Eftersom denna funktion är svår att hitta en primitiv funktion till så började jag med att försöka göra "jämförelsetestet". Så jag försökte först hitta en funktion som är större eller lika med funktionen i min integral. Jag valde funktionen g(x)=x3+1 och beräknade dess integral/gränsvärde:

limN0Nx3+1 dx =limN x44+ x = limNN44+N = 

Eftersom att integralen av g(x) inte konvergerar så konvergerar inte heller min första integral. Fine. Men när jag kollar i facit så står det att integralen i uppgiften faktiskt konvergerar men jämförelsetestet motsäger detta. Har testat "kvottestet" också men har bara fått 0 eller  som gränsvärde så det hjälper inte.

Så nu är jag helt vilse... hur fortsätter jag då? Vad gör jag nu?

SeriousCephalopod 2693
Postad: 1 okt 2018 16:55

När det kommer till rationella funktioner så är den prottypiska intregralen

1xαdx\int \frac{1}{x^\alpha}dx

där denna divergerar om exponentens är mindre än 2 α<2\alpha <> och konvergerar om exponenten är större än 2, α2\alpha \geq 2.

 

Så om du exempelvis ville visa att integralen konvergerar du bör undersöka är om du kan hitta ett

1xα\frac{1}{x^\alpha}

som som är större än din funktion och där exponenten är större än eller lika med 2 när x är tillräckligt stort. Dvs att din graf förr eller senare hamna under exempelvis 1/x^2 för då är arean garanterat ändlig. 

Nide 114
Postad: 1 okt 2018 17:03 Redigerad: 1 okt 2018 17:07
SeriousCephalopod skrev:

När det kommer till rationella funktioner så är den prottypiska intregralen

1xαdx\int \frac{1}{x^\alpha}dx

där denna divergerar om exponentens är mindre än 2 α<>\alpha <> och konvergerar om exponenten är större än 2, α2\alpha \geq 2.

 

Så om du exempelvis ville visa att integralen konvergerar du bör undersöka är om du kan hitta ett

1xα\frac{1}{x^\alpha}

som som är större än din funktion och där exponenten är större än eller lika med 2 när x är tillräckligt stort. Dvs att din graf förr eller senare hamna under exempelvis 1/x^2 för då är arean garanterat ändlig. 

 Aha. Var aldrig medveten om detta. Kommer detta från en viss sats eller...? Har den satsen ett namn?

EDIT: Testade att räkna ut integralen av 1/x^2 när x->oändligheten i Wolfram Alpha och Wolfram Alpha säger att integralen INTE konvergerar. Va...?

AlvinB 4014
Postad: 1 okt 2018 17:05 Redigerad: 1 okt 2018 17:12
SeriousCephalopod skrev:

När det kommer till rationella funktioner så är den prottypiska intregralen

1xαdx\int \frac{1}{x^\alpha}dx

där denna divergerar om exponentens är mindre än 2 α<>\alpha <> och konvergerar om exponenten är större än 2, α2\alpha \geq 2.

 

Så om du exempelvis ville visa att integralen konvergerar du bör undersöka är om du kan hitta ett

1xα\frac{1}{x^\alpha}

som som är större än din funktion och där exponenten är större än eller lika med 2 när x är tillräckligt stort. Dvs att din graf förr eller senare hamna under exempelvis 1/x^2 för då är arean garanterat ändlig. 

 Olikheterna skall väl ändå vara α1\alpha \leq 1 för divergens och α>1\alpha > 1 för konvergens då

11x dx\displaystyle \int_1^{\infty} \frac{1}{x}\ dx divergerar

medans

11x1,001 dx\displaystyle \int_1^{\infty} \frac{1}{x^{1,001}}\ dx konvergerar och är lika med 10001000.

EDIT:

Man kan visa varför detta gäller genom att beräkna integralen:

11xα dx=[1(1-α)xα-1]1\displaystyle \int_1^{\infty} \frac{1}{x^{\alpha}}\ dx=[\frac{1}{(1-\alpha)x^{\alpha-1}}]_1^{\infty}

Gränsvärdet för den övre integrationsgränsen konvergerar enbart om α-1>0\alpha-1>0 vilket ger α>1\alpha>1.

Nide 114
Postad: 1 okt 2018 17:11
AlvinB skrev:
SeriousCephalopod skrev:

När det kommer till rationella funktioner så är den prottypiska intregralen

1xαdx\int \frac{1}{x^\alpha}dx

där denna divergerar om exponentens är mindre än 2 α<>\alpha <> och konvergerar om exponenten är större än 2, α2\alpha \geq 2.

 

Så om du exempelvis ville visa att integralen konvergerar du bör undersöka är om du kan hitta ett

1xα\frac{1}{x^\alpha}

som som är större än din funktion och där exponenten är större än eller lika med 2 när x är tillräckligt stort. Dvs att din graf förr eller senare hamna under exempelvis 1/x^2 för då är arean garanterat ändlig. 

 Olikheterna skall väl ändå vara α1\alpha \leq 1 för divergens och α>1\alpha > 1 för konvergens då

11x dx\displaystyle \int_1^{\infty} \frac{1}{x}\ dx divergerar

medans

11x1,001 dx\displaystyle \int_1^{\infty} \frac{1}{x^{1,001}}\ dx konvergerar och är lika med 10001000.

 Detta låter mer logiskt, men hur tillämpar jag detta faktum för att bevisa att min integral konvergerar. Dem har ju inte samma intervall ([0,] mot [1, ]) eller spelar detta ens någon roll?

AlvinB 4014
Postad: 1 okt 2018 17:13

Dela upp integrationsintervallet i två - [0,1][0,1] och [1,][1,\infty]. Du ser snabbt att [0,1][0,1]-integralen konvergerar och därför kommer konvergensen att bero på den andra delen.

SeriousCephalopod 2693
Postad: 1 okt 2018 17:18 Redigerad: 1 okt 2018 17:19

Stämmer att mina olikheter blev knasifga med 1 och inte 2. Skrev fel.

Nide 114
Postad: 1 okt 2018 17:20
AlvinB skrev:

Dela upp integrationsintervallet i två - [0,1][0,1] och [1,][1,\infty]. Du ser snabbt att [0,1][0,1]-integralen konvergerar och därför kommer konvergensen att bero på den andra delen.

 Ahhh... såklart! Varför tänkte jag inte på det! Tack! :)

Albiki 5096 – Fd. Medlem
Postad: 1 okt 2018 17:40

Låt N>5N>5 och xNx\geq N. För sådana xx är integranden uppåt begränsad enligt

    (x3+1)/(x5+x3+1)(x3+x3)/(x5+x3)=2/(x2+1)2/x2(x^3+1)/(x^5+x^3+1)\leq (x^3+x^3)/(x^5+x^3) = 2/(x^2+1) \leq 2/x^2

Detta betyder att den givna integralen är konvergent. 

Svara Avbryt
Close